By taking the product between the number of shirts and pants, we coclude that there are 600 different outfit combinations.
How many outfit combinations are possible?
The total number of outfit combinations is given by the product between the numbers of each type of clothes that you have.
you have 40 shirts.
Yo have 15 pairs of pants.
Then the number of different combinations is 40*15 = 600
Learn more about combinations:
https://brainly.com/question/11732255
#SPJ1
What operation do you think Diego’s used to calculate the lengths for his drawing?
1. Diego subtract 10 to each length as follows:
15 - 10 = 5
36 - 10 = 26
30 - 10 = 20
18 - 10 = 8
12 - 10 = 2
21 - 10 = 11
In this way only the side which length is 5 is correct
2. Jada scales the drawing by the next scale factor:
5/15 = 1/3
where 5 is the desired length and 15 is the length of the original side.
Then, the other lengths are:
36*1/3 = 12
30*1/3 = 10
18*1/3 = 6
12*1/3 = 4
21*1/3 = 7
3. Diego's procedure is incorrect, as it is seen in his drawing. Subtraction is not the correct way to scale a drawing. On the other hand, Jada's procedure is correct.
Answer:
Step-by-step explanation:
Multiply 26 times 8 and then 26 times 20
So the answer is multiplication!
A boat rental service charges a $20 transportation fee and $30 dollars an
hour to rent a boat. Write an equation for the total cost for the day. Let C=
the total cost, and let H= number of hours. *
Answer:
c=2h+20
Step-by-step explanation:
2h reprents the totals of hours times the 30-dollar hour use.
the 20 is added because it's the transportation fee.
:
What is 475.189 rounded to the nearest hundredth?
Answer:
It is:475.19.
HOPE THIS HELPED
Answer:
475.19.
Step-by-step explanation:
The hundredths place is two places to the right of the decimal, or (0.01).
In the number given, the number to the right of the hundredths place is larger than 5, meaning we will round up giving us:
475.189 --> 475.19.
A triangle has sides with lengths of 6
centimeters, 11 centimeters, and 13
centimeters. Is it a right triangle? How
do you know?
Plss help
Answer:
no
Step-by-step explanation:
6^2 + 11^2 = 36 + 121 = 157
13^2 = 169
those are not equal so it is not a right triangle
Center: (2, 8) radius: 3
how do I write the equation of the circle in standard form?
We are given:
Coordinates of center: (2,8)
Radius of circle: 3 units
Finding equation of the circle:
We know that the general form of a circle is:
(x-h)² + (y-k)² = (radius)²
where h is the x-coordinate of the center and k is the y-coordinate
replacing with the given values, we get:
(x-2)² + (y-8)² = (3)²
help
plz (A) 4.00
(B) 0.25
(C) 1.00
(D)0.10
Answer:
0.25
Step-by-step explanation:
2.00÷8=0.25 and the rate keeps going constant at a rate of 0.25
Ray has football practice on Mondays Wednesdays and Fridays the clock below shows when the practice starts and ends how many minutes does Ray spend at the football practice each week
Answer:
is one of the answers 225?
Step-by-step explanation:
Answer:
225 is correct
Step-by-step explanation:
Practice is 75 minutes each day and Ray practices three days a week so multiply the time by three.
Find the equation of a circle centered at (3, -3) and passing through P(3, 0).
Answer:
(x - 3)² + (y + 3)² = 9
Step-by-step explanation:
the equation of a circle in standard form is
(x - h)² + (y - k)² = r²
where (h, k ) are the coordinates of the centre and r is the radius
the radius r is the distance from the centre to a point on the circle.
given (3, - 3 ) is the centre and (3, 0 ) is a point on the circle
since the x- coordinates of both points are 3 then r is the absolute value of the difference of the y- coordinates, that is
r = | - 3 - 0 | = | - 3 | = 3
then equation of circle is
(x - 3)² + (y - (- 3) )² = 3² , that is
(x - 3)² + (y + 3)² = 9
Juan sold a bicycle at a discount of 15%. If the selling price was $340, find the usual price of the bicycle.
Answer: $400
Step-by-step explanation:
Discount = 15%
The original price/value of an item is always 100%
So selling price (%) = original price - discount = 100%-15% = 85%
We got selling price as 85%
This implies that 85% = 340
Let's find 1% first, then 100%
1% = 340÷85 = 4
100% = 4 × 100 = $400
The usual (normal/original) price is $400
Juan sold a bicycle at a discount of 15% if the selling price was $340 then the usual price of the bicycle was $400.
What is Percentage?percentage, a relative value indicating hundredth parts of any quantity.
Let's represent the usual price of the bicycle by P.
Since Juan sold the bicycle at a discount of 15%, the selling price (S) would be 85% of the usual price (P).
We can express this relationship as an equation:
S = 0.85P
We also know that the selling price of the bicycle was $340.
Substituting S = $340 into the equation above, we get:
$340 = 0.85P
To find P, we can solve for it:
P = $340 / 0.85
P = $400
Therefore, the usual price of the bicycle was $400.
To learn more on Percentage click:
https://brainly.com/question/24159063
#SPJ2
A survey found that the American family generates an average of 17.2 pounds of glass garbage each year. Assume the standard deviation of the distribution is 2.5 pounds. Find the probability that the mean of a sample of 40 families will be between 17.1 and 18.1 pounds. Assume that the sample is taken from a large population and the correction factor can be ignored. Round your final
answer to four decimal places and intermediate z-value calculations to two decimal places
The probability that the mean of a sample of 40 families will be between 17.1 and 18.1 pounds is given as follows:
0.5874 = 58.74%.
How to obtain probabilities using the normal distribution?The z-score of a measure X of a variable that has mean symbolized by \(\mu\) and standard deviation symbolized by \(\sigma\) is obtained by the rule presented as follows:
\(Z = \frac{X - \mu}{\sigma}\)
The z-score represents how many standard deviations the measure X is above or below the mean of the distribution, depending if the obtained z-score is positive or negative.Using the z-score table, the p-value associated with the calculated z-score is found, and it represents the percentile of the measure X in the distribution.By the Central Limit Theorem, the sampling distribution of sample means of size n has standard deviation \(s = \frac{\sigma}{\sqrt{n}}\).The parameters for this problem are given as follows:
\(\mu = 17.2, \sigma = 2.5, n = 40, s = \frac{2.5}{\sqrt{40}} = 0.3953\)
The probability that the mean of a sample of 40 families will be between 17.1 and 18.1 pounds is the p-value of Z when X = 18.1 subtracted by the p-value of Z when X = 17.1, hence:
\(Z = \frac{X - \mu}{\sigma}\)
By the Central Limit Theorem:
\(Z = \frac{X - \mu}{s}\)
Z = (18.1 - 17.2)/0.3953
Z = 2.28
Z = 2.28 has a p-value of 0.9887.
Z = (17.1 - 17.2)/0.3953
Z = -0.25
Z = -0.25 has a p-value of 0.4013.
Hence:
0.9887 - 0.4013 = 0.5874 = 58.74%.
More can be learned about the normal distribution at https://brainly.com/question/25800303
#SPJ1
Use the Quadratic Formula to solve the equation x 2 − 23 = 10 x
Answer:
x = 5+4√3 or 5–4√3
Step-by-step explanation:
x² – 23 = 10x
x² – 10x –23 = 0
Using quadratic formula
a = 1, b = –10 and c = –23
x = –b ± (√b² – 4ac)/2a
–(–10) ± (√(-10)² – 4(1)(–23))/2(1)
=>10 ± (√100 + 92)/2
=> 10 ± (√192)/2
=> (10 ± 13.856)/2
x = (10 + 13.856)/2 or (10 – 13.856)/2
x = 11.828 or –1.928
x = 5+4√3 or 5–4√3
How many pounds are in 1.2 tons
Answer: 2,400
Step-by-step explanation: Multiply the mass value by 2000
Answer:
2400 pounds
Step-by-step explanation:
1 ton is 2000 pounds
So we can multiply the number of tons by 2000 to get the answer.
1.2 * 2000 = 2400 pounds
May I please have a Brainliest? I put a lot of thought and effort into my answers, so I would really appreciate it!
PLEASE HELP ASAP!!!! DUE TODAY WILL GIVE BRAINLIEST
If JKL is rotated 180° clockwise about the origin, what are the new coordinates of point J?
A (6,4)
B. (4, -6)
C.(6, -4)
D.(-4,-6)
Answer:
we conclude that if JKL is rotated 180° clockwise about the origin, what are the new coordinates of point J(-4, 6) will be: J' (4, -6)
Hence, the answer choice (B) is correct.
Step-by-step explanation:
The rule of rotating a point 180° clockwise about the origin states that if we rotate a point P(x, y) 180° clockwise about the origin, it would take a new position with the coordinates P'(-x, y).
In other words, the sign of its x and y coordinates change.
Thus, the rule is:
P(x, y) → P'(-x, -y)
Given the triangle ΔJKL with the coordinates
J(-4, 6)K(-1, 2)J(-4, 2)Thus, according to the rule:
P(x, y) → P'(-x, -y)The new position of point J (-4, 6) will be J' (4, -6)
The new position of point K (-1, 2) will be K' (1, -2)
The new position of point L (-4, 2) will be L' (4, -2)
Finally, we conclude that if JKL is rotated 180° clockwise about the origin, what are the new coordinates of point J(-4, 6) will be: J' (4, -6)
Hence, the answer choice (B) is correct.
HELPPPP!!! 30 POINTSSSSSS!!!!!!
Find the line of best fit for the set of data:
x y
-2 2.9
-3.5 2
1.4 4.8
-4.2 1.5
0 4
2.8 6
-1.5 3.5
A. y = .613x + 4.142
B. y = -.613x - 4.142
C. y = -.613x + 4.142
D. y = .613x - 4.142
Answer:
\(\boxed{\tt A. \;y = .613x + 4.142}\)
Step-by-step explanation:
Equation: (y-y_1)=y_2-y_1/x_2-x_1 (x-x_1)
Here,
\(\tt (x_1,y_1):(-2,2.9)\)
\(\tt (x_2,y_2): (-3.5,2)\)
\(\tt y-2.9=\cfrac{(2-2.9)}{(-3-5-(-2))} (x-(-2))\)
\(\tt y-2.9=\cfrac{-0.9}{-1.5} (x+2)\)
\(\tt y-2.9=0.6(x+2)\)
\(\tt y-2.9=0.6x+1.2\)
\(\tt y=.613x+4.142\)
___________________
Hope this helps you!
Have a nice day!
A grocery store has 10 cartons of yogurt for sale, of which 4 are raspberry. What is the probability that a randomly selected carton of yogurt will be raspberry? Write your answer as a fraction or whole number.
Answer:
2/5
Step-by-step explanation:
4 raspberry / 10 total
4/10
2/5
Almost all employees working for financial companies in New York City receive large bonuses at the end of the year. A sample of 62 employees selected from financial companies in New York City showed that they received an average bonus of $47,000 last year with a standard deviation of $15,000. Construct a 90% confidence interval for the average bonus that all employees working for financial companies in New York City received last year.
Answer:
The 90% confidence interval for the average bonus that all employees working for financial companies in New York City received last year is between $43,819 and $50,181
Step-by-step explanation:
We have the standard deviation for the sample, which means that the t-distribution is used to solve this question.
T interval
The first step to solve this problem is finding how many degrees of freedom, we have. This is the sample size subtracted by 1. So
df = 62 - 1 = 61
90% confidence interval
Now, we have to find a value of T, which is found looking at the t table, with 61 degrees of freedom(y-axis) and a confidence level of \(1 - \frac{1 - 0.9}{2} = 0.95\). So we have T = 1.67
The margin of error is:
\(M = T\frac{s}{\sqrt{n}} = 1.67\frac{15000}{\sqrt{62}} = 3181\)
In which s is the standard deviation of the sample and n is the size of the sample.
The lower end of the interval is the sample mean subtracted by M. So it is 47000 - 3181 = $43,819
The upper end of the interval is the sample mean added to M. So it is 47000 + 3181 = $50,181
The 90% confidence interval for the average bonus that all employees working for financial companies in New York City received last year is between $43,819 and $50,181
Find the probability that when a couple has four children, at least one of them is a boy. (Assume that boys and girls are equally likely.)
Answer:
The probability that at least, one of the four children the couple has is a boy is 0.8.
Step-by-step explanation:
Given that boys and girls are equally likely, we want to find the probability of having at least, one boy, from four children..
Note that it is possible to have the following for 4 children:
1. 4 boys, 0 girls
2. 3 boys, 1 girl
3. 2 boys, 2 girls
4. 1 boy, 3 girls
5. 0 boys, 4 girls.
To have at least, one boy, out of the 5 options, only 4 is possible.
1. 4 boys, 0 girls.........YES
2. 3 boys, 1 girl ...........YES
3. 2 boys, 2 girls.........YES
4. 1 boy, 3 girls.............YES
5. 0 boys, 4 girls..........NO
The probability is therefore,
(Probability of event = 4) ÷ (Total possible outcome = 5)
P = 4/5 = 0.8
(6.2 x 10²) x (3.5 x 10³)
Answer:
\(21.7 x 10^5\)
Step-by-step explanation:
(6.2 x 10²) x (3.5 x 10³)
First, multiply the coefficients: 6.2 x 3.5 = 21.7.
Then, add the exponents: 10² x 10³ = 10^(2+3) = 10^5.
Therefore, the result is 21.7 x 10^5.
Answer:
3286000
Step-by-step explanation:
A company packs its fruit salad in
containers that hold 1 5/8 pounds
How many containers does it
need to hold 585 pounds of fruit
salad?
Answer: 360 containers
Step-by-step explanation: 1 5/8 in decimal form is 1.625 pounds. Divide 585 by 1.625 pounds you get 360. It takes 360 containers to hold 585 pounds of fruit
PLEASE HELP ! Multiple answers
Answer:
i know for a fact B and C are correct im not so sure about a and d tho (i dont think A and D are right)
Step-by-step explanation:
There is a line of sym at o.5 because its in the middle of 0 and 1
f(x) is decreasing between -4 and 0 because f(x) or y is decreasing
Daisy cream is sold in a bulk of 76 cups of cream. Kremlin cream is sold in a bulk of 4 1/2 gallons of cream. Marble cream is sold in a bulk of 40 pints of cream. Which one has the most cream?
Therefore , the solution of the given problem of unitary method comes out to be Daisy cream and Kremlin cream both have less cream per bulk 1 gallon and 4.5 gallons, respectively than Marble cream.
An unitary method is what?The objective can be accomplished by utilising what has already been discovered, taking advantage of this worldwide access, and including all essential components from earlier changeable study who employed a certain technique. If the anticipated claim outcome actually occurs, it will either be possible to contact the variable again or both important processes will undoubtedly miss the statement.
Here,
We must convert cups to gallons because daisy cream is sold in bulks of cups. Since a gallon of Daisy cream comprises 16 cups, one quantity of Daisy cream contains:
=> 16 cups per bulk = 1 gallon 16 cups per bulk = 1 gallon
Moscow cream is offered in bulk quantities of 4 1/2 gallons, which is one gallon.
Thus, we must convert pints to gallons. A gallon of Marble cream comprises eight pints, hence one quantity of Marble cream contains:
=> 40 pints in a bulk equal 1 gallon, 8 pints, or 5 gallons.
As a result, we can see that Marble cream, with 5 gallons per bulk, has the most cream.
Daisy cream and Kremlin cream both have less cream per bulk (1 gallon and 4.5 gallons, respectively) than Marble cream.
To know more about unitary method visit:
https://brainly.com/question/28276953
#SPJ1
If you need any help with fractions I'm here to help. This is for 4th grade and less.
Just comment :)
Ok, thanks for your support, You're amazing!!!!! :)
Answer: wow ur rlly nice to offer ur service to ppl.
Step-by-step explanation:
Question 2 of 39
Look at this series: 56, 54, 50, 38, 34, ... What number should come next?
A) 23
B) 18
C) 20
D) 32
E) 26
Answer:
D) 32
Step-by-step explanation:
Given;
series: 56, 54, 50, 38, 34, ....
find the difference between successive numbers;
56 54 50 38 34 x
2 4 12 4 2
The highest difference between the numbers is 12, after which it decreases to 4 and finally back 2.
The next number "x" is calculated as;
34 - 2 = x
34 - 2 = x
32 = x
Therefore, the next number should be 32.
The next number that should come next in the series is 32.
Sequence is an arrangement of two or more things in a successive order. Series is the cumulative sum of a given sequence of terms.
Given the numbers in the series: 56, 54, 50, 38, 34, . . ..
The difference is 2, 4, 12, 4, 2
The highest difference between the numbers is 12, after which it decreases to 4 and finally back 2. Hence:
The next number = 34 - 2 = 32
The next number that should come next in the series is 32.
Find out more on series at: https://brainly.com/question/13171394
Graph the circle (x-2)^2 + (y-7)^2 = 4
Answer:
see attached
Step-by-step explanation:
You want the graph of the circle defined by the equation ...
(x-2)^2 + (y-7)^2 = 4.
Circle equationThe equation of a circle with center (h, k) and radius r is ...
(x -h)² +(y -k)² = r²
Comparing this to the given equation, we see ...
h = 2, k = 7, r² = 4
Then r = √4 = 2.
The circle will have its center at (2, 7) and will have a radius of 2. It is shown in the attached graph.
<95141404393>
3 1/2 inches so what is the area of this circle?
Answer:
19.6 in²
Step-by-step explanation:
area of circle = π r²
= π (3 1/2)²
= π (5/2)²
= π 25/4
= 22/7 x 25/4
= 275/14
=19.6 in²
please. help.....
.....
Answer:
1)
√50
50=2·5²
The 5s can be taken out to get 5√2
2)
√24
24=2³·3
Two 2s can be taken out to get 2√6
3)
√192
192=2⁶·3
We can take the 2s out to get 8√3
4)
√169
169=13²
The 13s can be taken out to get 13
5)
√147
147=3·7²
The 7s can be taken out to get 7√3
6)
√(13/49)
√13/√49
13 is prime, 49=7²
(√13)/7
7)
√(6/27)
√6/√27
6=2·3 and 27=3³
Two 3s can be taken out to get √6/3√3
√6 can be rewritten as √2·√3
In (√2·√3)/3√3 the √3s cancel leaving √2/3
8)
3/√6
In fractions you can multiply and divide by x/x
In this case, x=√6
3√6/(√6·√6)
3√6/6
6=3·2 and the 3s cancel out leaving √6/2
Record Examination) are normally distributed with a mean of 555 and a standard
deviation of 110. Use the 68-95-99.7 Rule to find the percentage of people taking the test who score below 335.
The percentage of people taking the test who score below 335 is
Answer:
2.5%
Step-by-step explanation:
You want the percentage below 335 if the distribution is normal with a mean of 555 and a standard deviation of 110, using the empirical rule.
Z scoreThe z-score of 335 is ...
Z = (X -µ)/σ
Z = (335 -555)/110 = -220/110 = -2
DistributionThe empirical rule tells you that 95% of the distribution is between Z = -2 and Z = 2. That is, 5% of the distribution is evenly split between the tails Z < -2 and Z > 2. Half that value is in each tail.
P(X < 335) = 5%/2 = 2.5%
The percentage of people taking the test who score below 335 is 2.5%.
<95141404393>
How many solutions exist for the system of equations below?
[3x+y=18
13x+y=16
Answer:
1 solution
Step-by-step explanation:
The two equations have different slopes and different y-intercepts, so they represent two lines that intersect at 1 point.
Answer: 1 solution
sin( 3pi/4 ) =
O A. 1/2
OB. -√2/2
O C. √3/2
O D. √2/2
Answer:
sin( 3pi/4 ) = -√2/2
So, B.
Two people are planning their wedding. For the reception, they found the the cost C for 50 guests, g is $2150 whereas the cost for 75 guests is $3025. Calculate the slope to find the cost per guest?
The slope which shows the cost per guest is $35 per guest
The given cost for 50 guests is $2150.
The cost for 75 guests is $3025.
It can also be represented as:
(Guest, Cost) =(50, $2150) & (75, $3025)
The slope can now be calculated as
Slope = (y₂ - y₁)/(x₂ - x₁)
Where
(x, y) = (50, $2150) & (75, $3025)
Substituting the given values in the equation as follows:
Slope = (3025 - 2150)/(75 - 50)
Slope = 35
Hence, the slope is $35 per guest.
To know more on Slope click:
https://brainly.com/question/29481191
#SPJ1